4
$\begingroup$

Let $F$ be a perfect field, and let $p(t) \in F[t]$ be an irreducible monic polynomial of degree $n$ such that: $$p(t) = t^n+s_1 t^{n-1}+s_2 t^{n-2}+\dots+s_n$$ Let $\theta_1, \theta_2, \dots, \theta_n$ be the roots of $p(t)$ in some fixed algebraic closure $\overline{F}$. Consider the polynomial ring in $n$ indeterminates, $R = F[x_1, x_2, \dots, x_n]$. Define the Viète ideal $\mathcal{V}_p \subset R$ as the ideal generated by the elementary symmetric functions evaluated at the coefficients of $p$: $$\mathcal{V}_p=\langle e_1(\vec{x})-s_1,e_2(\vec{x})-s_2,\dots,e_n(\vec{x})-s_n \rangle$$ where $$e_1(\vec{x}) = x_1 + x_2 + \dots + x_n$$ $$e_2(\vec{x}) = \sum_{1 \le i < j \le n} x_i x_j$$ $$\vdots$$ $$e_n(\vec{x}) = x_1 x_2 \cdots x_n$$ In this setting, a point $(a_1, \dots, a_n) \in \overline{F}^n$ is a solution to the system defined by $\mathcal{V}_p$ if and only if $\{a_1, \dots, a_n\} = \{\theta_1, \dots, \theta_n\}$ as sets. Moreover, the quotient ring $A = R / \mathcal{V}_p$ encodes all the symmetric relations of the roots.

My Question: How can one explicitly describe and calculate the maximal ideals $\mathfrak{m}$ of the ring $A$?

I am particularly interested in understanding how the Galois group of $p$ arises as a subgroup of the automorphism group of $A$. The latter holds, because for any maximal ideal $M\supseteq \mathcal{V}_p$, $$F(\theta_1,...,\theta_n)\cong R/M\cong (R/\mathcal{V}_p)/(M/\mathcal{V}_p)$$

I would greatly appreciate any insights, references, or explicit calculations for this problem.


Insights to the problem:

  • It may be useful to have in account that, for some ordering $\mathbf{r}=(\theta_1,...,\theta_n)$ of the roots, maximal ideals should have the form $$\mathfrak{m}_{\mathbf{r}}=⟨x_1-\theta_1,...,x_n-\theta_n⟩\cap R$$ so that $\mathfrak{m}_{\mathbf{r}}=\{f\in R|f(\theta_1,...,\theta_n)=0\}$
  • As per observed with Qiaochu Yuan's characterization, the ideal $\mathcal{V}_p$ is itself maximal in $R$ iff $\text{Gal}(p)\cong S_n$. This case results in the obvious conclussion that $\mathfrak{m}=⟨0⟩$ in the ring $A$.
$\endgroup$
1
  • $\begingroup$ I deleted my answer, my bad. My argument would only hold if the sets $\{f\in R|f(\theta_1,...,\theta_n)=0\}$ and $\{f\in R|f(\theta_{\sigma(1)},...,\theta_{\sigma(n)});\ \sigma\in S_n\}$ are equals... But the former is generally a superset of the latter. $\endgroup$ Commented Aug 26 at 19:40

1 Answer 1

8
$\begingroup$

This is not really going to be an answer to your question but here is a long comment. It's a little more conceptual to rewrite the defining relations of $A$ in the form

$$A = F[x_1, \dots x_n]/(p(t) = \prod (t - x_i))$$

where $t$ is not an element of $A$ but shorthand for saying that we are imposing exactly the relations implied by the polynomial identity $p(t) = \prod (t - x_i)$, which has the effect of "universally splitting" $p$. Our relations imply that $p(x_1) = 0$, hence that the subalgebra generated by $x_1$ is $A_1 = F[x_1]/p(x_1)$, which has dimension $n$. Working over this subalgebra, we can divide $p(t)$ by $t - x_1$, giving a polynomial identity of the form

$$\frac{p(t)}{t - x_1} = \prod_{i \neq 1} (t - x_i)$$

where the LHS and hence the RHS lives in $A_1[t]$. Writing $p_1(t)$ for the LHS, this gives $p_1(x_2) = 0$, hence the subalgebra generated by $x_2$ over $A_1$ is (possibly a quotient of) $A_2 = A_1[x_2]/p_1(x_2)$, which has dimension at most $n-1$.

Continuing in this way we find that $A$ is spanned as a vector space by the monomials $x_1^{i_1} x_2^{i_2} \dots x_n^{i_n}$ where $0 \le i_j \le j-1$, and so has dimension at most $n!$.

Now let $L$ be the splitting field generated inside $\overline{F}$ by the roots $\theta_i$, and consider the extension of scalars $A_L = A \otimes_F L$. By construction there are exactly $n!$ $L$-homomorphisms $A_L \to L$, corresponding to bijections between the $x_i$ and the $\theta_i$, each of which has kernel a distinct maximal ideal $(x_i - \theta_{\sigma(i)})$. This gives a homomorphism

$$A_L \to \prod_{\sigma \in S_n} L$$

which is surjective by the Chinese remainder theorem, and since $\dim_L A_L \le n!$ it must be an isomorphism. So $\dim_F A = n!$ and $A_L$ is isomorphic to a product

$$\boxed{ A_L \cong \prod_{\sigma \in S_n} L }$$

of $n!$ copies of $L$.

To recover $A$ from $A_L$ we apply Galois descent: $A$ is the fixed subalgebra of $A_L$ under the action of the Galois group $G = \text{Gal}(L/F)$ on $A_L$ given by its action on the right tensor factor $L$ of $A \otimes_F L$. Thinking of $G$ as a subgroup of $S_n$ acting on the $\theta_i$ by permutation, if $g \in G$ sends $\theta_i$ to $\theta_{g(i)}$ then it sends the maximal ideal $(x_i - \theta_{\sigma(i)})$ to $(x_i - \theta_{g(\sigma(i))})$ (possibly I need to insert some inverses here but I am going to blithely ignore this because it doesn't affect the substance of the argument). So $G$ acts by permuting the maximal ideals. The orbits of this action can be identified (non-canonically) with the $\frac{n!}{|G|}$ cosets of $G$ inside $S_n$, and taking fixed points has the effect of collapsing each orbit into a single copy of $L$; the result is that $A$ is isomorphic to a product

$$\boxed{ A \cong \prod_{\sigma \in S_n/G} L }$$

of $\frac{n!}{|G|}$ copies of $L$. This is of course compatible with the observation that by construction there are $n!$ $F$-homomorphisms $A \to L$, which have the same kernel if they are related by the action of $G$. The above argument is a somewhat indirect way to show that the kernels of these homomorphisms account for all maximal ideals of $A$ and that their intersection is zero.

Now for the bad news: this doesn't really help us explicitly compute the maximal ideals of $A$. It's already clear that these are exactly the kernels of the homomorphisms $A \to L$ (this follows e.g. from the Nullstellensatz), so we just get that the elements of these ideals are polynomial identities satisfied by the roots $\theta_i$, and I don't know any general results about how to compute these. Consider for example the case $F = \mathbb{Q}, p = \Phi_n$ of the cyclotomic polynomials; here we can take the $\theta_i$ to be the primitive $n^{th}$ roots of unity in $\mathbb{C}$, which satisfy many "incidental" relations, and I don't see any obvious way to deduce these relations directly from $p$ or the Galois group. A similar example where there will be many "incidental" relations occurs if we take $F$ to be a finite field; generally there will be more incidental relations the smaller the Galois group $G$ (and hence the splitting field $L$) is.

I guess one potential approach is to consider the increasing sequence $L_i = F[\theta_1, \dots \theta_i] \subseteq L$ and try to compute the corresponding sequence of minimal polynomials of $\theta_i$ over $L_{i-1}$. I don't know how easy this is to work or compute with in practice, though.

$\endgroup$
7
  • 1
    $\begingroup$ Your product description shows that the automorphism group of $A$ is the wreath product $G \wr S_{n!/|G|}$, which contains $G$ as a subgroup in many different ways (if $|G|<n!$), which I guess answers the final part of the question. $\endgroup$ Commented Aug 25 at 17:44
  • 1
    $\begingroup$ @Will: I didn't want to comment on this because I wasn't sure what the cleanest way of describing what's going on here is. The problem is that if we specifically define $G = \text{Gal}(L/F)$ with $L$ as above, then the isomorphism between each quotient $A/m$ by a maximal ideal and $L$ is, of course, not canonical. Additionally there is an obvious (canonical) action of $S_n$ on $A$ given by permuting the $x_i$, and some subgroup of this corresponds to some copy of $G$, but which subgroup depends up to conjugacy on which bijection is chosen between the $x_i$ and the $\theta_i$. $\endgroup$ Commented Aug 25 at 18:40
  • 2
    $\begingroup$ When a group $H$ acts transitively on a set $X$, one can consider the group whose elements consist of a permutation $\sigma$ of $X$, and, for each $x\in X$, a $g_x\in H$ with $g_x\cdot x = \sigma(x)$. This is noncanonically isomorphic to the wreath product of $S_X$ with the stabilizer of an element in $H$. $\endgroup$ Commented Aug 25 at 19:08
  • 1
    $\begingroup$ Taking $H=S_n$ and $X = S_n/G$, I think this gives the right description, since an automorphism induces a permutation $\sigma$ of the set of maximal ideals of $A$ and, for each maximal ideal $\mathfrak m$, an isomorphism from $A/\mathfrak m$ to $A/ \sigma(\mathfrak m)$, where the images of $x_1,\dots,x_n$ in $A/\mathfrak m$ are necessarily sent to the images of $x_1,\dots, x_n$ in $A/ \sigma(\mathfrak m)$ under some permutation in $S_n=H$. $\endgroup$ Commented Aug 25 at 19:09
  • $\begingroup$ This is a truly convenient description, thanks! From this result, it follows trivially that $\mathcal{V}_p$ is maximal iff $\text{Gal}(p)\cong S_n$ $\endgroup$ Commented Aug 26 at 10:55

You must log in to answer this question.

Start asking to get answers

Find the answer to your question by asking.

Ask question

Explore related questions

See similar questions with these tags.